The function h is given by h(x)=log_2(x^2 -6). For what positive value of x does h(x)=4?

Answers

Answer 1

The function h is given by h(x)=log2(x² -6). The positive value of x that makes h(x) equal to 4 is approximately 4.69

We have the function:

h(x) = log2(x² - 6)

We want to find the value of x that makes h(x) equal to 4:

h(x) = 4

log2(x² - 6) = 4

We can rewrite this equation as:

2⁴ = x² - 6

16 = x² - 6

x²= 22

x = √22 (because we are looking for a positive value of x)

Therefore, the positive value of x that makes h(x) equal to 4 is approximately 4.69 (rounded to two decimal places).

Learn more about logarithm function at https://brainly.com/question/4563843

#SPJ11


Related Questions

3. The scale of a room in a blueprint is 2 inches : 1 foot. A window in the same blueprint is 12 inches. Complete the table. Blueprint Length (in.) Actual Length (ft) a. How long is the actual window? 2 1 4 3 4 10 12 5 6 b. A mantel in the room has an actual width of 8 feet. What is the width of the mantel in the blueprint?​

Answers

Therefor, the length of mantel in blueprint is > 30 ft

width of the mantel in the blueprint 8ft×2inc/1ft=16inch

what is width?

The term "width" refers to the length from side to side of anything. For instance, the shorter side of a rectangle would be the width.

we know that

[scale]=[blueprint]/[actual]-------> [actual]=[blueprint]/[scale]

[scale]=3/5 in/ft

for [wall blueprint]=18 in

[wall actual]=[wall blueprint]/[scale]-------> 18/(3/5)----> 30 ft

Part A)

the actual wall is 30 ft  long

Part B) window has actual width of 2.5 ft

[ window blueprint]=[scale]*[actual window]-----> (3/5)*2.5----> 1.5 in

the width of the window in the blueprint is 1.5 in

Part C) Complete the table

For [blueprint length]=4 in

[actual length]=[blueprint length]/[scale]-------> 4/(3/5)----> 20/3 ft

For [blueprint length]=5 in

[actual length]=[blueprint length]/[scale]-------> 5/(3/5)----> 25/3 ft

For [blueprint length]=6 in

[actual length]=[blueprint length]/[scale]-------> 6/(3/5)----> 30/3=10 ft

For [blueprint length]=7 in

[actual length]=[blueprint length]/[scale]-------> 7/(3/5)----> 35/3 ft

For [actual length]=6 ft

[blueprint length]=[actual length]*[scale]-------> 6*(3/5)----> 18/5 in

For [actual length]=7 ft

[blueprint length]=[actual length]*[scale]-------> 7*(3/5)----> 21/5 in

For [actual length]=8 ft

[blueprint length]=[actual length]*[scale]-------> 8*(3/5)----> 24/5 in

For [actual length]=9 ft

[blueprint length]=[actual length]*[scale]-------> 9*(3/5)----> 27/5 in

B) width of the mantel in the blueprint 8ft×2inc/1ft=16inch

To know more about width visit:

brainly.com/question/30173060

#SPJ1

Solve each system by substitution
Y=-7x-24
Y=-2x-4

Answers

Answer:

(- 4, 4 )

Step-by-step explanation:

y = - 7x - 24 → (1)

y = - 2x - 4 → (2)

substitute y = - 2x - 4 into (1)

- 2x - 4 = - 7x - 24 ( add 7x to both sides )

5x - 4 = - 24 ( add 4 to both sides )

5x = - 20 ( divide both sides by 5 )

x = - 4

substitute x = - 4 into either of the 2 equations and evaluate for y

substituting into (1)

y = - 7(- 4) - 24 = 28 - 24 = 4

solution is (- 4, 4 )

The cone is formed from 3,200 ft3 of gravel. If the height of the cone is 24 feet, what is the radius, in feet, of the base of the cone? Use the π button on your calculator to determine the answer. Round your answer to the nearest tenth of afoot. The radius of the base of the cone is approximately ____ feet

Answers

The radius of the base of the cone is approximately 12.65 feet if The cone is formed from 3,200 ft of gravel.

Height of cone =  24 feet

The volume of the cone = [tex]3,200 ft^3[/tex]

To find the volume of the cone, the formula used here is:

V =π* [tex]r^2h[/tex]

Here, the values of V and H are known terms. we need to calculate the radius r of the cone.

π = 3.14 constant value

Substituting the values in the above equation, we get:

[tex]3,200 = (1/3)^2*(24)*[/tex] π

[tex]r^2[/tex]= 3,200 / (8π)

[tex]r^2[/tex] = 400 / π

r = 12.65

Therefore, we can conclude that the radius of the base of the cone is  12.65 feet.

To learn more about Volume of Cone

https://brainly.com/question/29767724

#SPJ4

A force of 80 pounds on a rope is used to pull a box up a ramp inclined at 10 degrees from the horizontal. The rope forms an angle of 33 degrees with the horizontal. How much work is done pulling the box 26 feet along the ramp?

Answers

The work done on the displacement is 301.95J

What is the work done in pulling the box

To determine the work done, we need to find the displacement in which the box moved.

cos θ = adjacent / hypothenuse

cos 33 = adjacent / 80

adjacent = 80 * cos 33

adjacent = 67.1 lbs

The force applied is 67.1lbs

The displacement on the ramp;

sin θ = opposite / hypothenuse

sin 10 = opposite / 26

opposite = 26 * sin 10

opposite = 4.5 ft

The work done in moving the object can be calculated as;

work done = force * displacement

work done = 67.1 * 4.5

work done = 301.95 J

Learn more on work done here;

https://brainly.com/question/25573309

#SPJ1

8. (02.03 mc)


costs of attendance


category


dollar amount


annual tuition and fees


$4,934.00


annual room and board


$1,424.00


annual cost of books and supplies $1,250.00


other one-time fee


$275.00


annual scholarship and grants


$5,250.00


using the information from the table, identify the equation in slope-intercept form that models the total cost of attendance. (1 point)


o y = 2,358x + 275


o y = 2,633x


o y = 7,608x + 275


o y = 7,883

Answers

The equation in slope-intercept form that models the total cost of attendance is: y = 2,633x + 275.


1. Add up the annual costs: tuition and fees ($4,934), room and board ($1,424), and cost of books and supplies ($1,250) to get the total annual cost: $4,934 + $1,424 + $1,250 = $7,608.


2. Subtract the annual scholarship and grants from the total annual cost: $7,608 - $5,250 = $2,358. This is the slope (x) of the equation, as it represents the cost per year.


3. The other one-time fee ($275) is the y-intercept of the equation, as it's a fixed cost that does not change with the number of years.


4. Put the slope and y-intercept into the slope-intercept form (y = mx + b) to get: y = 2,633x + 275.

To know more about slope-intercept form click on below link:

https://brainly.com/question/29146348#

#SPJ11

Let S be the part of the plane 3+ + 2) + z = 1 which lies in the first octant, oriented upward. Use the Stokes theorem to find the flux of the vector field F = 3i+3j + 4k across the surface S.

Answers

The surface integral of the dot product between the vector field F = 3i + 3j + 4k and the unit normal vector of the surface S is equal to zero.

To use Stokes' theorem to find the flux of the vector field F = 3i + 3j + 4k across the surface S, which is the part of the plane 3x + 2y + z = 1 in the first octant and oriented upward.

Stoke's theorem statement is “the surface integral of the curl of a function over the surface bounded by a closed surface will be equal to the line integral of the particular vector function around it.” Stokes theorem gives a relation between line integrals and surface integrals.

First, we need to parametirize the curve C that bounds the surface S. Since S is in the first octant, x, y, and z are all non-negative.

The boundary C consists of three line segments: (i) from (0, 0, 0) to (1/3, 0, 0), (ii) from (1/3, 0, 0) to (0, 1/2, 0), and (iii) from (0, 1/2, 0) to (0, 0, 0). Next, calculate the curl of F, which is the cross product of the del operator and F:
curl(F) = (∂Fz/∂y - ∂Fy/∂z)i - (∂Fx/∂z - ∂Fz/∂x)j + (∂Fy/∂x - ∂Fx/∂y)k = (0 - 0)i - (0 - 0)j + (0 - 0)k = 0.

Since curl(F) = 0, the line integral of F over C is also 0.

According to Stokes' theorem, the flux of F across S equals the line integral of F over C, which we found to be 0.

Therefore, the flux of the vector field F = 3i + 3j + 4k across the surface S is 0.

Learn More About Stoke's Theoram: https://brainly.com/question/17256782

#SPJ11

James weighs 8712 pounds. he has 2 dogs that each weigh 1314 pounds. how many more pounds does james weigh than both of his dogs combined?

Answers

James weighs 6084 more pounds than both of his dogs combined.

To find out how many more pounds James weighs than both of his dogs combined, we first need to calculate the total weight of the dogs. Since he has two dogs that weigh 1314 pounds each, we can find the total weight of the dogs by multiplying 1314 by 2, which gives us 2628 pounds.

Next, we can add the weight of both dogs together to get the total weight of the dogs, which is 2628 pounds. We can then subtract the weight of the dogs (2628 pounds) from James' weight (8712 pounds) to find out how many more pounds James weighs than both of his dogs combined.

Therefore, James weighs 6084 more pounds than both of his dogs combined. This can be calculated by subtracting the weight of the dogs (2628 pounds) from James' weight (8712 pounds), which gives us 6084 pounds.

To know more about weight, refer to the link below:

https://brainly.com/question/8308591#

#SPJ11

Resume the totat revenue from the sale of them is given by R(x) * 25 1n (6x + 1), while the total cost to produce x items is C(x)=ſ. Find the approximate number of items that should be manufactured so that profit, RIX-C) is maximum G A 143 Rems OB. 84 items C. 47 items OD 114 items

Answers

The approximate number of items that should be manufactured so that the profit, P(x) = R(x) - C(x), is maximum is 47 items (option C).

To find the approximate number of items that should be manufactured to maximize profit, we need to first find the profit function P(x) by subtracting the total cost, C(x), from the total revenue, R(x). Then, we need to find the critical points of P(x) and determine which one corresponds to the maximum profit.

process of finding profit:

Step 1: Find the profit function P(x) = R(x) - C(x)
Given R(x) = 25 ln(6x + 1) and C(x) = ∫x, let's find P(x):
P(x) = R(x) - C(x)
P(x) = 25 ln(6x + 1) - ∫x
Step 2: Find the critical points of P(x)
To find the critical points, we need to take the derivative of P(x) and set it equal to 0:
P'(x) = d/dx [25 ln(6x + 1) - ∫x]
Since the derivative of ln(6x + 1) is (6/(6x + 1)), and the derivative of ∫x is x:
P'(x) = 25 [tex]\times[/tex] (6/(6x + 1)) - x
Now, set P'(x) = 0 and solve for x:
25 [tex]\times[/tex] (6/(6x + 1)) - x = 0
Step 3: Determine which critical point corresponds to the maximum profit
The approximate number of items that should be manufactured so that the profit, P(x) = R(x) - C(x), is maximum is 47 items (option C).

To know more about Profit:

https://brainly.com/question/15036999

#SPJ11

Can someone help me please

Answers

Answer:

A

Step-by-step explanation:

Simple interest gains interest only on the principal sum and so each year has the same interest. So, this example is not simple interest. Compound interest is calculated on the principal and the accumulated interest of the previous years. So, this is compound interest.

      Interest is 4%.

      [tex]A = P*(1 + R \%)^n[/tex]

Here A is the amount we receive after n years, P is the principal and R is the rate of interest.

        A = $1300 , R = 4%

            [tex]1300 = P * (1+0.04})\\\\1300 = P * 1.04\\\\\dfrac{1300}{1.04}=P\\\\\\\dfrac{130000}{104}=P[/tex]

            P = $ 1250

The point (7,8) in the coordinate plane represents a ratio. adela claims that you can find equivalent ratio by adding the same number to both coordinate of the point. is adela correct? explain.

Answers

For the point (7,8) in the coordinate plane which represents a ratio, Adela claims that you can find equivalent ratio by adding the same number to both coordinate of the point is incorrect.

Adela claim is not correct. To find an equivalent ratio, you should multiply (or divide) both coordinates by the same nonzero number instead of adding the same number.

1. The point (7,8) represents the ratio 7:8.

2. If we add the same number to both coordinates, let's say 2, we get the point (9,10), which represents the ratio 9:10.

3. We can check if 7:8 and 9:10 are equivalent ratios by cross-multiplying:

7 * 10 = 70 and 8 * 9 = 72. Since 70 ≠ 72, these ratios are not equivalent.

Therefore, Adela's claim is incorrect because adding the same number to both coordinates of the point does not result in an equivalent ratio. To find equivalent ratios, you should multiply (or divide) both coordinates by the same nonzero number.

Learn more about Equivalent ratio:

https://brainly.com/question/2914376

#SPJ11

143°
(8x+55)° find the value of X

Answers

Answer:11

Step-by-step explanation:Vertically opposite angles are the same.
Alternate(Z)angles are the same.Therefore we have an angle of 143 vertically opposite to the equation.143-55=88
88/8=11

Consider the function f(x) = 2x³ + 6x² – 144x + 4, -6 ≤ x ≤ 5. Find the absolute minimum value of this function. Answer: Find the absolute maximum value of this function. Answer:

Answers

The absolute maximum value of the function f(x) is 222.

To find the absolute minimum value of the function f(x), we need to first find the critical points within the given interval -6 ≤ x ≤ 5. To do this, we take the derivative of f(x) and set it equal to zero:

f'(x) = 6x² + 12x - 144
0 = 6(x² + 2x - 24)
0 = 6(x+6)(x-4)

The critical points are x=-6, x=-4, and x=4. To determine which of these points correspond to a minimum value, we evaluate f(x) at each of these points and at the endpoints of the interval:

f(-6) = -880, f(-4) = -184, f(4) = -136, f(-6) = -880, f(5) = 222

Therefore, the absolute minimum value of the function f(x) is -880.

To find the absolute maximum value of the function f(x), we follow the same process. The critical points are still x=-6, x=-4, and x=4, but now we need to evaluate f(x) at each of these points and at the endpoints of the interval to determine which corresponds to a maximum value:

f(-6) = -880, f(-4) = -184, f(4) = -136, f(-6) = -880, f(5) = 222

Therefore, the absolute maximum value of the function f(x) is 222.

To learn more about critical points, refer below:

https://brainly.com/question/7805334

#SPJ11

What is the median of the lower half of data

Answers

We can see here that in order to find the median of the lower half of data, one will have to sort out the data in an ascending order. Then take the  lower half of the data (i.e., the first half of the sorted data) and find its median.

What is median?

The median, which is used to measure central tendency in statistics, is the point at which a dataset may be divided into two equal parts. If a dataset has an even number of values, it is the average of the two middle values or the middle value in a sorted dataset.

The values in the dataset must first be arranged from lowest to highest in order to determine the median. The median is the middle value if the dataset has an odd number of values.

Learn more about median on https://brainly.com/question/26177250

#SPJ1

Determine the equation of the tangent plane to the surface z = y ln x at the point (1, 4, 0)

Answers

z = 4(x - 1) - ln (y - 4)

Therefore, the equation of the tangent plane to the surface z = y ln x at the point (1, 4, 0) is z = 4(x - 1) - ln (y - 4).

To determine the equation of the tangent plane to the surface z = y ln x at the point (1, 4, 0), we first need to find the partial derivatives of the surface with respect to x and y.

∂z/∂x = y/x

∂z/∂y = ln x

Then, we can use these partial derivatives along with the point (1, 4, 0) to find the equation of the tangent plane using the formula:

z - z0 = ∂z/∂x(x0, y0)(x - x0) + ∂z/∂y(x0, y0)(y - y0)

where (x0, y0, z0) is the given point.

Plugging in the values, we have:

z - 0 = (4/1)(x - 1) + ln 1(y - 4)

Simplifying:

z = 4(x - 1) - ln (y - 4)

Therefore, the equation of the tangent plane to the surface z = y ln x at the point (1, 4, 0) is z = 4(x - 1) - ln (y - 4).

Learn more about tangent plane here:

https://brainly.com/question/30260323

#SPJ11

Every time you practice, you gain more skills.



Conditional:


Hypothesis:


Conclusion:


Converse:


Inverse:


Contrapositive:

Answers

Hypothesis: Every time you practice, you gain more skills.

What happens when you practice?

Conclusion: Gain of skills is a result of practice.

Converse: If you gain more skills, then you practice every time.

Inverse: If you don't practice, then you don't gain more skills.

Contrapositive: If you don't gain more skills, then you don't practice every time.

The hypothesis states that practicing leads to an increase in skills. This can be interpreted as a cause and effect relationship between the two variables.

The conclusion reiterates that gaining skills is a result of practice.

The converse of the statement flips the order of the hypothesis and the conclusion. It states that if you gain more skills, then you must have practiced every time.

This may not be entirely true because there can be other factors that contribute to the gain of skills besides practice.

The inverse of the statement negates both the hypothesis and the conclusion. It states that if you don't practice, then you don't gain more skills.

This statement is true because practice is a necessary condition for gaining skills. However, it doesn't mean that practicing alone guarantees the gain of skills.

The contrapositive of the statement flips the order of the negated hypothesis and the negated conclusion. It states that if you don't gain more skills, then you didn't practice every time.

This statement is also true because if one doesn't practice, they cannot expect to gain more skills.

Learn more about practice

brainly.com/question/2962699

#SPJ11

Tell whether the angles are adjacent or vertical. Then find the value of x. Please help with this question

Answers

Answers - adjacent angles, x = 63 degrees

Explanation

Adjacent angles share a side, vertical angles are across from each other so this is adjacent

This is a straight angle that includes these adjacent angles, a straight angle sum is 180 degrees.

180 - 117 = 63

X = 63 degrees
Answers - adjacent angles, × = 63 degrees

help i need this done pls 50 points

Answers

The length of the diagonal is 12. 7in

How to determine the length

To determine the length of the diagonal, we need to know the Pythagorean theorem.

The Pythagorean theorem states that the square of the hypotenuse side is equal to the sum of the squares of the other two sides of a triangle.

The other two sides are the opposite and the adjacent sides.

From the information given in the diagram, we have that;

The opposite side = 12in

The adjacent side = 4in

Substitute the values

x² = 12² + 4²

find the squares

x² = 160

find the square root

x = 12. 7 in

Learn about Pythagorean theorem at: https://brainly.com/question/654982

#SPJ1

Find the error & explain why it is wrong:
megan solved the following problem. what did she do wrong?
what is (f - g)(2)?
f(x) = 3x2 – 2x + 4
g(x) = x2 – 5x + 2

Answers

The value of (f-g)(2) is 16, provided that Megan has made no mistakes in the calculation.

Find the error in the given problem solved by Megan?

The problem asks us to compute the value of (f - g)(2) where f(x) = 3x^2 - 2x + 4 and g(x) = x^2 - 5x + 2.

The notation (f - g)(2) means that we need to subtract g(x) from f(x) and then evaluate the result at x = 2. We can do this as follows:

(f - g)(x) = f(x) - g(x) = (3x^2 - 2x + 4) - (x^2 - 5x + 2) = 2x^2 + 3x + 2

Substituting x = 2, we get:

(f - g)(2) = 2(2)^2 + 3(2) + 2 = 16

Therefore, the value of (f - g)(2) is 16.

It's worth noting that the problem statement mentions "what did she do wrong?" without providing any context or information about what Megan did or didn't do. So, it's not possible to identify any error in Megan's solution based on the given information. However, based on the correct computation above, we can be sure that (f - g)(2) is indeed equal to 16.

In other words, it can be described as,

The error in Megan's solution is not clear from the given statement. However, it seems that she may have made an error while computing (f-g)(2).

To compute (f-g)(2), we need to subtract g(2) from f(2) as follows:

f(2) = 3(2)^2 - 2(2) + 4 = 12

g(2) = (2)^2 - 5(2) + 2 = -4

Therefore, (f-g)(2) = f(2) - g(2) = 12 - (-4) = 16. is the final conclusion.

Learn more about Solution

brainly.com/question/30198131

#SPJ11

Suppose a bus arrives at a bus stop every 26 minutes. If you arrive at the bus stop at a random time, what is the probability that you will have to wait at least 4 minutes for the bus?

Answers

The time between buses arriving at the stop follows an exponential distribution with a mean of 26 minutes.

To find the probability of waiting at least 4 minutes for the bus, we can use the cumulative distribution function (CDF) of the exponential distribution:

P(waiting at least 4 minutes) = 1 - P(waiting less than 4 minutes)

The probability of waiting less than 4 minutes can be calculated using the CDF:

P(waiting less than 4 minutes) = 1 - e^(-4/26) ≈ 0.146

Therefore, the probability of waiting at least 4 minutes for the bus is:

P(waiting at least 4 minutes) = 1 - 0.146 ≈ 0.854

So the probability of having to wait at least 4 minutes for the bus is about 85.4%.

Let f:R → R be a function that satisfies ∫f(t)dt then the value of f(log e 5) is

Answers

Unfortunately, I cannot provide an answer to this question as it is incomplete. The given information ∫f(t)dt is not enough to determine the value of f(log e 5). More information about the function f would be needed, such as its explicit form or additional properties. Please provide more context or information to help me answer your question accurately.
Given that f is a function f:R → R that satisfies ∫f(t)dt, we need to find the value of f(log e 5).

By definition, log e 5 is the natural logarithm of 5, which can be written as ln(5). Therefore, we want to find the value of f(ln(5)).

However, without further information on the function f or the integral bounds, it's not possible to determine the exact value of f(ln(5)). Please provide more details about the function or the integral to get a specific answer.

Learn more about integration here: brainly.com/question/18125359

#SPJ11

When solving two-step equations, you are using the reverse order of operations to solve the two-step equations.

Select one:
True
False

Answers

Therefore , the solution of the given problem of equation comes out to be False.

What is an equation?

In order to demonstrate consistency between two opposing statements, variable words are frequently used in sophisticated algorithms. Equations are academic phrases that are used to demonstrate the equality of different academic figures. Consider expression the details as y + 7 offers. In this case, elevating produces b + 7 when partnered with building y + 7.

Here,

False.

The order of operations we use to solve two-step equations is the same order we use to solve every other mathematical statement,

which is commonly recalled by the acronym PEMDAS. (parentheses, exponents, multiplication and division from left to right, and addition and subtraction from left to right).

The fundamental distinction is that we are carrying out the operations against the equation's representation. For instance, consider the following equation:

=> 2x + 5 = 11

To get the following result, we would first subtract 5 from both sides, then divide by 2.

=> x = 3

In order to "undo" the operations that were carried out on the variable in the original equation, we are utilising the same series of operations as usual, but in reverse order.

To know more about equation visit:

https://brainly.com/question/649785

#SPJ1

expand and simplify(2w-3)3

Answers

Answer:

6w-9

Step-by-step explanation:

2w×3=6w

-3×3=-9

=6w-9

A health insurance company wants to know the proportion of admitted hospital patients who have a type 2 diabetes at orlando health hospitals. how large of a sample should be taken to estimate the proportion within 6% at 93% confidence?

Answers

To estimate the proportion of admitted hospital patients who have type 2 diabetes at Orlando Health Hospitals within a certain margin of error and confidence level,

we can use the following formula to determine the necessary sample size:

[tex]n = [(Z^2 * p * q) / E^2] / [1 + ((Z^2 * p * q) / E^2N)][/tex]

where:

n = sample size

Z = z-score for the desired confidence level (in this case, 1.81 for a 93% confidence level)

p = estimated proportion of patients with type 2 diabetes (unknown)

q = 1 - p

E = margin of error (0.06)

N = population size (unknown)

Since we do not know the estimated proportion of patients with type 2 diabetes or the population size,

we can assume a conservative estimate of p = q = 0.5, which maximizes the sample size required.

Plugging in the values, we get:

[tex]n = [(1.81^2 * 0.5 * 0.5) / 0.06^2] / [1 + ((1.81^2 * 0.5 * 0.5) / 0.06^2N)][/tex]

Simplifying, we get:

[tex]n = 1242.95 / [1 + (2.4 / N)][/tex]

To satisfy the requirements of the problem, we need to round up to the nearest whole number, so we need a sample size of at least 1243 patients.

Note that if we had a better estimate for p or N, we could use those values in the formula to get a more precise sample size.

To know more about margin of error refer here

https://brainly.com/question/29101642#

#SPJ11

Find f'(4) for f(x) 8/In(3x^2) Round to 3 decimal places, if necessary.

Answers

To find f'(4), we need to take the derivative of f(x) with respect to x and then evaluate it at x=4. Using the chain rule, we get:
f'(x) = -16x/(ln(3x^2))^2

So, f'(4) = -16(4)/(ln(3(4)^2))^2 = -64/(ln(48))^2

Rounding to 3 decimal places, we get f'(4) = -0.019.
To find f'(4) for f(x) = 8/ln(3x^2), we first need to differentiate f(x) with respect to x. We will use the quotient rule and the chain rule for this purpose.

The quotient rule states: (u/v)' = (u'v - uv')/v^2, where u = 8 and v = ln(3x^2).

Now, differentiate u and v with respect to x:
u' = 0 (since 8 is a constant)
v' = d(ln(3x^2))/dx = (1/(3x^2)) * d(3x^2)/dx (using chain rule)

Now, differentiate 3x^2 with respect to x:
d(3x^2)/dx = 6x

So, v' = (1/(3x^2)) * (6x) = 2/x

Now, apply the quotient rule for f'(x):
f'(x) = (0 - 8 * (2/x))/(ln(3x^2))^2 = -16/(x * (ln(3x^2))^2)

Now, plug in x = 4 to find f'(4):
f'(4) = -16/(4 * (ln(3*(4^2)))^2) = -16/(4 * (ln(48))^2)

Rounded to 3 decimal places, f'(4) ≈ -0.171.

Visit here to learn more about derivative  : https://brainly.com/question/25324584
#SPJ11

For y = 72√x, find dy, given x = 4 and Δx = dx = 0.21
dy = (Simplify your answer.)

Answers

To find dy for the function y = 72√x, given x = 4 and Δx = dx = 0.21, we will first find the derivative of y with respect to x and then plug in the given values.

1. Differentiate y with respect to x: y = 72√x can be rewritten as y = 72x^(1/2)
  Apply the power rule: dy/dx = 72 * (1/2)x^(-1/2)
  Simplify: dy/dx = 36x^(-1/2)

2. Plug in the given values: x = 4 and dx = 0.21
  dy/dx = 36(4)^(-1/2)
  dy/dx = 36(1/√4)
  dy/dx = 36(1/2)
  dy/dx = 18

3. Calculate dy: dy = (dy/dx) * dx
  dy = 18 * 0.21
  dy = 3.78

So, for y = 72√x, dy is 3.78 when x = 4 and Δx = dx = 0.21.

To learn more about ''find dy, given x'' visit : https://brainly.com/question/15272082

#SPJ11

Water began leaking from a holes in a bucket at a constant rate. Here is a Table of how many ounces were in the bucket at

different times. 10 am:

304 ounces

Noon :

228 ounces

3 pm :

114 ounces


there are more then one question to answer


What is the amount of water that leaks from the bucket each hour?


How many ounces of water were in the bucket at 8 am. ?\


At what time will the bucket be empty. ?


Write the equation of this function in slope intercept form. Use x for the amount of time in hours since the leak began and y for the amount of water in the buck

Answers

The amount of water that leaks from the bucket each hour is 38 ounces/hour.

The amount of water in the bucket at 8 am would have been 380 ounces. The bucket will be empty after 8 hours, or at 6 pm.

To find the amount of water that leaks from the bucket each hour, we can use the information that the leak is at a constant rate. We can find the total amount of water that leaked out of the bucket from 10 am to 3 pm, which is 304 - 114 = 190 ounces.

This is over a time period of 5 hours (from 10 am to 3 pm), so the amount of water that leaks from the bucket each hour is:

190 ounces ÷ 5 hours = 38 ounces/hour

To find how many ounces of water were in the bucket at 8 am, we need to estimate the amount of water that leaked out from 8 am to 10 am. We know that the bucket loses 38 ounces of water every hour, so from 8 am to 10 am (2 hours), the amount of water that leaked out would be:

38 ounces/hour x 2 hours = 76 ounces

Therefore, the amount of water in the bucket at 8 am would have been:

304 ounces + 76 ounces = 380 ounces

To find at what time the bucket will be empty, we can assume that the leak rate remains constant at 38 ounces/hour. We know that the bucket starts with 304 ounces, so we can set up the equation:

y = 304 - 38x

where y is the amount of water in the bucket and x is the time in hours since the leak began. When the bucket is empty, y will be zero, so we can solve for x:

0 = 304 - 38x

38x = 304

x = 8

Therefore, the bucket will be empty after 8 hours, or at 6 pm.

The equation for the amount of water in the bucket as a function of time can be written in slope-intercept form as:

y = -38x + 304

where the slope (m) is -38 (the rate at which water is leaking out of the bucket) and the y-intercept (b) is 304 (the initial amount of water in the bucket).

To know more about equation, refer to the link below:

https://brainly.com/question/29987959#

#SPJ11

Solve the equation and check your solution: x + 4 = -2 + x

Answers

The equation x + 4 = -2 + x has no solution for x

Solving the equation and checking the solution

From the question, we have the following parameters that can be used in our computation:

x + 4 = -2 + x

Subtract x from both sides of the equation

so, we have the following representation

x - x + 4 = -2 + x - x

When the like terms of the equation are evaluated, we have

4 = -2

The above equation is false

This is because 4 and -2 do not have the same value

Hence, the equation has no solution

Read more about equation at

https://brainly.com/question/18831322

#SPJ1

Which expressions represent the length of side MN?



Choose 2 answers:



A. 4 • sin (65)



B. 1. 9 • cos (65)



C. 4/sin (65)



D. 1. 9/cos (65)



PLEASE HELP!!

Answers

Expressions A and B represent the length of side MN.

How can the length of side MN be expressed?

The two expressions that represent the length of side MN are A) 4•sin(65) and B) 1.9•cos(65). The length of side MN can be found using the trigonometric ratios of sine and cosine in a right triangle.

The angle of 65 degrees is opposite to side MN and the hypotenuse of the triangle is given as 4 units. Using the sine ratio, we can find the length of MN as 4•sin(65). Similarly, using the cosine ratio, we can find the length of MN as 1.9•cos(65).

Therefore, expressions A and B both represent the length of side MN, and they have been obtained by using different trigonometric ratios.

Learn more about expressions

brainly.com/question/14083225

#SPJ11

Consider the circle centered at the origin and passing through the point (0, 4)

Answers

Equation of the circle: x^2 + (y - 2)^2 = 4

How to find the equation of the circle?

The circle centered at the origin and passing through the point (0, 4) can be represented by the equation of a circle. The general equation of a circle is (x - h)^2 + (y - k)^2 = r^2, where (h, k) represents the center of the circle and r represents the radius.

Since the center is at the origin (0, 0), the equation simplifies to x^2 + y^2 = r^2. To determine the radius, we can use the point (0, 4) that lies on the circle. Substituting these coordinates into the equation, we get 0^2 + 4^2 = r^2. Simplifying, we find that 16 = r^2.

Therefore, the equation of the circle centered at the origin and passing through the point (0, 4) is x^2 + y^2 = 16.

Learn about more origin

brainly.com/question/24281628

#SPJ11

• Orhan studied the relationship between


temperature and sales of refreshments


at the concession stands inside the football


stadium. He wrote an equation for the


linear function that relates temperature (x)


and refreshment sales (y). Which of the


following could be Orhan's equation?


A. Y=3x2 + 25


B. Y = 15x + 40


C. Y= llx - 55


-


D. Y= x – 135

Answers

The equation that could be Orhan's equation for the linear function that relates temperature and refreshment sales is   Y = 15x + 40.

This is because the equation is in the form of y = mx + b, where m is the slope (or rate of change) and b is the y-intercept. In this case, the slope is 15, which means that for every increase of 1 degree in temperature, there will be an increase of 15 units in refreshment sales.

The y-intercept is 40, which means that even at a temperature of 0 degrees, there will still be some refreshment sales (40 units).

The other equations do not have a linear relationship between temperature and sales, as they either have a quadratic term (A), a negative slope (C), or a large negative constant term (D).

Hence, option B is the correct answer.

To know more about Orhan's equation refer here:

https://brainly.com/question/27465710

#SPJ11

Other Questions
Does it seem natural for Linda and John to tell their histories to Bernard and Lenina when they first meet? What other ways might Huxley have chosen to help us understand these characters? Would it have worked if Linda had kept a journal that Bernard reads to Lenina? Or if the Indian guide explained what had happened to Linda? Can you think of another way? In bullet points what led to the empowerment of N Kanaka iwi which, in turn, led to the rise of the Hawaiian Renaissance. What happens to the frequency of the mass spring system if length of the spring iscut into one third Find the frequency of the 5th harmonic (h5) of a 5 hz fundamental Which of the following processes is used most often to amend the Constitution?G2/3 or more of state legislatures request a national convention, delegates propose an amendment, citizens elect representatives to send tostate conventions to consider the amendment, at least 3/4 of the conventions approve the amendment.Congress proposes an amendment, 2/3 of each house approves the amendment, 3/4 of state legislatures approve the amendment.Congress proposes an amendment, citizens elect representatives to send to state conventions to consider the amendment, at least 3/4 of theconventions approve the amendment.Congress proposes an amendment, 2/3 of each house approves the amendment, citizens elect representatives to send to state conventions toconsider the amendment, at least 3/4 of the conventions approve the amendment.092/3 or more of spate legislatures request a national convention, delegates propose an amendment, 3/4 of state legislatures approve theamendment.NextApr 278:10 Which correctly describes how to graph the equation shown below?y=1/4xStart with a point at (1, 4). Then go up 1 and 4 to the right.Start with a point at (1, 4). Then go up 4 and 1 to the right.Start with a point at (0, 0). Then go up 4 and 1 to the right.Start with a point at (0, 0). Then go up 1 and 4 to the right. Evaluate the integral by making an appropriate change of variables. Sle 9e2x + 2y da, where R is given by the inequality 2[x] + 2 y = 2 Consider the household model that you have seen in class but now assume that the goal of the household is to consume twice as much in period 2 as in period 1. she earns $100 in the first period and $150 in the second period. the interest rate is 5%. what is her optimal saving in the first period? A plate falls vertically to the floor and breaks up into three pieces, which slide along the floor. Immediately after the impact, a 320-g piece moves along the x-axis with a speed of 2. 00 m/s and a 355-g piece moves along the y-axis with a speed of 1. 50 m/s. The third piece has a mass of 100 g. In what direction does the third piece move? you can neglect any horizontal forces during the crash. Dunkin Donuts sent 20,311 emails including a link to a web page where customers could order a dozen birthday donuts. Of these, 452 emails were blocked or sent to bad addresses. This email campaign produced 1,784 visits to the web page. Calculate Clickthrough Rate for this email campaign. Report as a percent. Report 25.5%, for example, as "25.5". Rounding: tenth of a percent. Which of these best explains the mood in the first paragraph ofthe passage?O gloomy, because it tells about a difficult journeylighthearted, because it details a humorous exchangemysterious, because it describes an unfamiliarrationtriumphant, because it celebrates an important accomplishment 35 POINTS PLSSS HELPPWhat has the creation of professional teams (or sponsors, analysts etc.) meant for the development of eSports as an industry, where do you think the industry would be without them? In what way would things be different if games were played by individual players instead of teams? Garden plots in the Portland Community Garden are rectangles limited to 45 square meters. Christopher and his friends want a plot that has a width of 7.5 meters. What length will give a plot that has the maximum area allowed? Toy cars of different masses accelerate as they move down a ramp as shown in the figure. A motion sensor is used to measure the speed of each car. It is found that all of the cars have the same speed at the bottom of the ramp. This graph shows how the mass of the car affects the cars kinetic energy at the bottom of the ramp. Based on the graph, describe the relationship between the mass and the kinetic energy of the car, and then predict, in joules the kinetic energy of a car with a mass of 80 grams. People were asked if they were considering changing what they eat.29% of the people asked said yes.of these, 23% said they were considering becoming vegetarian.what percentage of the people asked said they were considering becoming vegetarian? 10. Determine the line of regression and use it to find the best predicted new mileage rating of a ( point) four-wheel-drive vehicle given that the old rating is 19 mi/gal old 6 27 1 33 28 24 8 22 20 29 21 New 15 24 15 29 25 22 6 20 826 19 Oy 0.808 0.863x; 18.1 mi/gal Oy = 0.863 + 0.808x; 16.2 mi/gal oy 0.863 + 0.808x; 22.4 mi/gal y-0.808+ 0.863x; 17.2 mi/gal Drag each label to the correct location on the diagram. Each label can be used more than once.What is the correct way to represent the ionic compound sodium fluoride?thank you so much Find the divergence of vector fields at all points where they are defined.div ( (2x^2 - sin(x2)) i + 5] - (sin(X2)) k) A telephone pole has a wire attached to its top that is anchored to the ground. the distance from the bottom of the pole to the anchor point is69 feet less than the height of the pole. if the wire is to be6 feet longer than the height of the pole, what is the height of the pole? In a recent Game Show Network survey, 30% of 5000 viewers are under 30. What is the margin of error at the 99% confidence interval? Using statistical terminology and a complete sentence, what does this mean? (Use z*=2. 576)Margin of error: Interpretation: